LSAT and Law School Admissions Forum

Get expert LSAT preparation and law school admissions advice from PowerScore Test Preparation.

 Administrator
PowerScore Staff
  • PowerScore Staff
  • Posts: 8917
  • Joined: Feb 02, 2011
|
#40387
Complete Question Explanation
(The complete setup for this game can be found here: lsat/viewtopic.php?t=13080)

The correct answer choice is (E)

This is the easiest question in this game, perhaps the test as a whole. If Q is fourth, and either Q or V must be third, then clearly V must be third. This validates answer choice (E).

Answer choice (A): This answer choice is incorrect, because R can be shown second.

Answer choice (B): This answer choice is incorrect, because R can be shown first.

Answer choice (C): This answer choice is incorrect, because S can be shown first or second.

Answer choice (D): This answer choice is incorrect because T must be shown fifth whenever V is shown third (see Templates 2.1. and 2.2. in the setup).

Answer choice (E): This is the correct answer choice. See Templates 2.1 and 2.2.

Get the most out of your LSAT Prep Plus subscription.

Analyze and track your performance with our Testing and Analytics Package.